Jump to content

Waiting for you's Content

There have been 33 items by Waiting for you (Search limited from 06-06-2020)



Sort by                Order  

#373890 Mỗi tuần một ca khúc!

Posted by Waiting for you on 29-11-2012 - 22:45 in Quán nhạc

thêm 1 bài cho 1 ngày buồn :angry:
Flower letter



#373809 \sum \frac{1}{a^2+bc}\leq \frac{...

Posted by Waiting for you on 29-11-2012 - 21:02 in Bất đẳng thức và cực trị

Cho $a,b,c$ là 3 cạnh của 1 tam giác
Chứng minh : $\sum \frac{1}{a^2+bc}\leq \frac{a+b+c}{2abc}$ ?

Phải chăng cách chứng minh này sai?
Áp dụng AM-GM cho mẫu
$\sum \frac{1}{a^2+2bc}\leq \sum \frac{1}{2a\sqrt{bc}}= \sum \frac{\sqrt{bc}}{2abc}\leq \sum \frac{\frac{1}{2}(b+c)}{2abc}= \frac{a+b+c}{2abc}$
Đấu bằng xảy ra khi tam giác trên đều hoặc suy biến thỏa mãn a=b=c?



#373399 BĐT AM-GM

Posted by Waiting for you on 28-11-2012 - 21:04 in Bất đẳng thức và cực trị

Mình xin đóng góp vài bài:
1) Cho $a,b,c>0$ chứng minh rằng
$\frac{ab}{c^{2}}+\frac{ca}{b^{2}}+\frac{bc}{a^{2}}\geq \frac{1}{2}(\frac{a+b}{c}+\frac{c+a}{b}+\frac{b+c}{a})$
2)Cho $a,b,c>0$ chứng minh rằng
$\frac{b+c}{a^{2}}+\frac{c+a}{b^{2}}+\frac{a+b}{c^{2}}\geq 2(\frac{1}{a}+\frac{1}{b}+\frac{1}{c})$
3)Cho $a,b,c>0$ thoả mãn $a+b+c=3$chứng minh rằng
$abc(a^{2}+b^{2}+c^{2})\leq 3$
4)Cho $a,b,c,d>0$ chứng minh rằng

$\frac{a-b}{b-c}+\frac{b-c}{c+d}+\frac{c-d}{d+a}+\frac{d-a}{a+b}\geq 0$

P/s: lời giải post sau ^^

Bài 1 hình như ta dùng đánh giá sau
Theo AM-GM thì $\frac{ab}{c^2}+\frac{ab}{c^2}+\frac{bc}{a^2}+\frac{ab}{c^2}+\frac{ab}{c^2}+\frac{ca}{b^2}\geq 3\frac{a+b}{c}$
Xây dựng các BĐT tương tự
Bài 2:Tách mỗi hạng tử vế trái thành 2 và sử dụng đánh giá
$\frac{b}{a^2}+\frac{1}{b}\geq \frac{2}{a}$
Bài 3:Có $abc(a^2+b^2+c^2)= \frac{1}{3}abc(a+b+c)(a^2+b^2+c^2)\leq \frac{1}{9}(ab+bc+ca)^2(a^2+b^2+c^2)$
Theo AM-GM tiếp thì $(ab+bc+ca)^2(a^2+b^2+c^2)=(ab+bc+ca)(ab+bc+ca)(a^2+b^2+c^2)\leq \frac{1}{27}(a+b+c)^3=27$
đây đã là dpcm
Bài 4:hình như là 1 BĐT của vasile?



#373351 Cho x,y,z>0. CMR: $xy+yz+zx\geq 2(x+y+z)$

Posted by Waiting for you on 28-11-2012 - 19:59 in Bất đẳng thức và cực trị

Cho x,y,z>0. CMR: $xy+yz+zx\geq 2(x+y+z)$

Đây dĩ nhiên là 1 BĐT sai?ngay với x=y=z=1,ta đã thấy???



#373327 Chứng minh rằng: $\sum \frac{1}{a\sqrt...

Posted by Waiting for you on 28-11-2012 - 19:15 in Bất đẳng thức và cực trị

Mọi người thử xem lời giải này xem sao
(minhtuyb) http://diendantoanho..._20#entry373195



#373322 CM:$\sum\frac{a^2}{b^3+c^3} \geq...

Posted by Waiting for you on 28-11-2012 - 19:08 in Bất đẳng thức - Cực trị

Chứng minh với mọi a,b,c dương có tổng bình phương bằng 3
$\frac{a^2}{b^3+c^3}+\frac{b^2}{c^3+a^3}+\frac{c^2}{a^3+b^3}\geq \frac{3\sqrt[3]{3}}{2\sqrt[3]{a^2+b^2+c^2}}$



#373134 BĐT AM-GM

Posted by Waiting for you on 27-11-2012 - 21:12 in Bất đẳng thức và cực trị

Chủ topic sao không thấy post bài nữa nhỉ? :wacko:



#373124 $a^4+b^4+c^4\geq a^3+b^3+c^3$

Posted by Waiting for you on 27-11-2012 - 20:54 in Bất đẳng thức và cực trị

Cho a,b,c dương có tổng bằng 3,chứng minh $a^4+b^4+c^4\geq a^3+b^3+c^3$



#373086 BĐT AM-GM

Posted by Waiting for you on 27-11-2012 - 19:00 in Bất đẳng thức và cực trị

Mình góp bài nhé
Cho $a,b,c>0$ và $a+b+c=3$.Chứng minh rằng
$a^2+b^2+c^2+\dfrac{ab+bc+ac}{a^2b+b^2c+c^2a} \ge 4$

minhtuyb said:Đề nghị các bạn giải quyết hết bài tồn động trước khi quăng bài mới :ph34r:
no matter what said:tạm thời mong mọi người tham gia XỬ LÝ các bài trong topic trước,khi nào phần kiến thức post hết,mọi người hãy thoải mái post bài ngoài.Mình thật sự xin lỗi vì sự bất tiện này và cũng mong mọi người cùng giúp để topic phát triển -Xin chân thành cảm ơn
mình nghĩ ta cứ làm như vậy đã nhỉ??? :icon6:



#373014 BĐT AM-GM

Posted by Waiting for you on 27-11-2012 - 13:00 in Bất đẳng thức và cực trị

Đề nghị các bạn giải quyết hết bài tồn động trước khi quăng bài mới :ph34r:
---
$$\dfrac{a^4}{a^2(b+c)}+\dfrac{a}{2}+\dfrac{a}{2}+\dfrac{b+c}{4}\ge 2a$$
Tương tự rồi cộng lại




Bài này mà $AM-GM$ được thì cũng hơi xoắn :o
---
$$\frac{1}{a(2b+2c-1)}=\dfrac{1}{a(1-2a)}$$
Sau đó ta đi c/m BĐT phụ:
$$\dfrac{1}{a(1-2a)}\ge 27a\\ \Leftrightarrow (3a-1)^2(6a+1)\ge 0$$

Phải chăng còn 1 cách bằng AM-GM kiểu này :wub:
Áp dụng trực tiếp AM-GM cho 3 số ta có
$VT\geq \frac{3}{\sqrt[3]{abc(2b+2c-1)(2c+2a-1)(2a+2b-1)}}$
Vậy ta chỉ cần chứng minh
$9\sqrt[3]{abc(2b+2c-1)(2c+2a-1)(2a+2b-1)}\leq 1$
tuy nhiên có thể thấy BĐT này đúng theo AM-GM
$\sqrt[3]{abc}\leq \frac{a+b+c}{3}=\frac{1}{3}$
$\sqrt[3]{(2b+2c-1)(2c+2a-1)(2a+2b-1)}\leq \frac{4(a+b+c)-3}{3}=\frac{1}{3}$



#372920 $\left\{\begin{matrix} x^3 +y^3=3xy+1...

Posted by Waiting for you on 26-11-2012 - 21:58 in Phương trình - hệ phương trình - bất phương trình

có lẽ pt trên là $...-1$ thì có khi còn có hi vọng

+ thêm điều kiện x,y không âm nữa,không thì bác AM-GM nỏ làm dc chi cả? :wacko:



#372915 $a+b+c=1$. Cmr : $b+c\geq 19abc$

Posted by Waiting for you on 26-11-2012 - 21:54 in Bất đẳng thức và cực trị

Cho $a,b,c$ không âm thỏa mãn : $a+b+c=1$. Cmr : $b+c\geq 19abc$

Đề bài cho a,b,c dương có tổng bằng 1,chứng minh $b+c\geq 16abc$
Giải?Ta có thể giải khá gọn bằng AM-GM
$1=(a+b+c)^2\geq 4a(b+c)\Rightarrow b+c\geq 4a(b+c)^2$
lại có $(b+c)^2\geq 2bc\Rightarrow 4a(b+c)^2\geq 16abc$
Bạn xem vậy có đúng không nhé :icon6:



#372910 $\left\{\begin{matrix} x^3 +y^3=3xy+1...

Posted by Waiting for you on 26-11-2012 - 21:49 in Phương trình - hệ phương trình - bất phương trình

Giải hpt:
$\left\{\begin{matrix} x^3 +y^3=3xy+1 & & \\ x^{2004}+y^{2004}=\frac{1}{2^{2003}} & & \end{matrix}\right.$
_______________________________________
@tramy : Chú ý cách đặt tiêu đề nhé bạn. :)

phải chi bài này sai đề nhỉ :icon6:
rõ ràng 2 pt của hệ không liên quan đến nhau và không thể đánh giá được phương trình dưới
Ta chỉ có 1 cách nghĩ là đánh giá phương trình trên (nhưng làm sao đánh giá được phương trình đó nhỉ?) :angry:



#372905 Cho $a, b, c >0$ và $ab+bc+ca=abc$. Chứng minh rằng:...

Posted by Waiting for you on 26-11-2012 - 21:42 in Bất đẳng thức và cực trị

Bài này có thể làm bằng phương pháp tọa độ...
Ai làm tiếp nhé

có phải ý tưởng tương tự bài này không bạn :wacko:
http://diendantoanho...85/#entry372708



#372900 $\sqrt{x^4+x^2+2x+1}+\sqrt{2x-5}+\sqr...

Posted by Waiting for you on 26-11-2012 - 21:36 in Phương trình, hệ phương trình và bất phương trình

Ngay khi giải điều kiện đã ra vô nghiệm rồi, bạn xem lại đề xem sao nhé.
đkxđ : $x^4+x^2+2x+1\geq 0$
$2x-5\geq 0\Rightarrow x\geq 2,5$ (1)
$2-x\geq 0\Rightarrow x\leq 2$ (2)
Thấy rõ (1) và (2) mâu thuẫn $\Rightarrow$ pt vô nghiệm.

đề hoàn toàn đúng :icon6: ,dĩ nhiên cách giải trên cũng không sai :(



#372883 $a\sqrt{a^{2}+2bc}+b\sqrt{b^{2...

Posted by Waiting for you on 26-11-2012 - 21:17 in Bất đẳng thức và cực trị

1. Cho a,b,c>0. C/m:
$a\sqrt{a^{2}+2bc}+b\sqrt{b^{2}+2ca}+c\sqrt{c^{2}+2ab}\geq \sqrt{3}(ab+bc+ca)$

Tiêu đề vi phạm nội quy bạn ơi tham khảo tại đây http://diendantoanho...eq-sqrt3abbcca/



#372881 $\sqrt{x^4+x^2+2x+1}+\sqrt{2x-5}+\sqr...

Posted by Waiting for you on 26-11-2012 - 21:13 in Phương trình, hệ phương trình và bất phương trình

Giải phương trình sau :lol:
$\sqrt{x^4+x^2+2x+1}+\sqrt{2x-5}+\sqrt{2-x}=0$



#372877 BĐT AM-GM

Posted by Waiting for you on 26-11-2012 - 21:08 in Bất đẳng thức và cực trị

Em đóng góp một bất đẳng thức nhỏ.
Chứng minh rằng với mọi số thực dương a,b,c thỏa mãn điều kiện a+b+c=1 thì:
$\sqrt {\frac{{ab}}{{c + ab}}} + \sqrt {\frac{{bc}}{{a + bc}}} + \sqrt {\frac{{ab}}{{c + ab}}} \le \frac{3}{2}$

Với điều kiện a+b+c=1,theo AM-GM,ta có
$\sqrt{\frac{ab}{c+ab}}=\sqrt{\frac{ab}{c(a+b+c)+ab}}= \sqrt{\frac{a.b}{(c+a).(c+b)}}\leq \frac{1}{2}(\frac{a}{c+a}+\frac{b}{c+b})$
Xây dựng các BĐT tương tự ta có dpcm



#371861 2.Chứng minh $\sum \sqrt{\frac{a^{2}...

Posted by Waiting for you on 23-11-2012 - 19:48 in Bất đẳng thức và cực trị

2. Cho a,b,c là các số thực dương thỏa mãn : $a^{2}+b^{2}+c^{2}=3$
Chứng minh :
$\sum \sqrt{\frac{a^{2}}{a^{2}+b+c}} \leq \sqrt{3}$

Ukraine 2008
Giải:Áp dụng C-S cho mẫu $(a^2+b+c)(1+b+c)\geq (a+b+c)^2$
Suy ra $\sqrt{\frac{a^2}{a^2+b+c}}\leq \frac{a\sqrt{1+b+c}}{a+b+c}$
Xây dsựng các BĐT tương tự,ta có
VT$\leq \frac{a\sqrt{1+b+c}+b\sqrt{1+c+a}+c\sqrt{1+a+b}}{a+b+c}$
Áp dụng C-S lần nữa $\frac{a\sqrt{1+b+c}+b\sqrt{1+c+a}+c\sqrt{1+a+b}}{a+b+c}\leq$
$\frac{\sqrt{(a+b+c)\left [ a(1+b+c)+b(1+c+a)+c(1+a+b) \right ]}}{a+b+c}$
=$\frac{\sqrt{(a+b+c)^2+(a+b+c)2(ab+bc+ca)}}{a+b+c}$
=$\sqrt{1+\frac{2(ab+bc+ca)}{a+b+c}}\leq \sqrt{1+\frac{2(a+b+c)}{3}}\leq \sqrt{1+\frac{2\sqrt{3(a^2+b^2+c^2)}}{3}}= \sqrt{3}$



#355142 $\frac{1}{a}+\frac{1}{b}+\frac{1}{c}+\frac{9}{a+b+c}...

Posted by Waiting for you on 18-09-2012 - 20:12 in Bất đẳng thức và cực trị

xin lỗi mình làm ngược dấu rồi



#355111 $\frac{1}{a}+\frac{1}{b}+\frac{1}{c}+\frac{9}{a+b+c}...

Posted by Waiting for you on 18-09-2012 - 18:43 in Bất đẳng thức và cực trị

Bài 1: Cho $x,y,z\epsilon \left [ 1,2 \right ] CMR: \left ( x+y+z \right )(\frac{1}{x}+\frac{1}{y}+\frac{1}{z})\geq 6(\frac{x}{y+z}+\frac{y}{x+z}+\frac{z}{x+y})$
Bài 2:Cho a,b,c>0.CM:
$\frac{1}{a}+\frac{1}{b}+\frac{1}{c}+\frac{9}{a+b+c}\geq 4(\frac{1}{b+c}+\frac{1}{c+a}+\frac{1}{a+b})$
ặc ai vào viết hộ em chữ $ vào cuối tiêu đề vs quên mất

củ tỏi thật nhnf đề này lại nhớ đến lần mình bị mắng vì không làm được bổ đề cơ bản của nó,mình xin giải bài 2
Bổ đề ,với a,b,c thực dương ,ta có BĐT sau
$\frac{b+c}{a}+\frac{c+a}{b}+\frac{a+b}{c}\geq 4(\frac{a}{b+c}+\frac{b}{c+a}+\frac{c}{a+b})$
(chứng minh bổ đề trên khá đơn giản bằng nhận xét theo AM-GM sau
$\frac{1}{a}+\frac{1}{b}\geq \frac{4}{a+b}$(hình như cũng có ở box THCS rồi ,bạn tham khảo lời giải chi tiết ở đó nhé)
trở lại bài toán ,GS $a\geq b\geq c\Rightarrow \frac{1}{b+c}\geq \frac{1}{c+a}\geq\frac{1}{a+b}$
tư tưởng của chúng ta sẽ là dùng chebyshev
$4\sum \frac{a}{b+c}\geq \frac{1}{3}(a+b+c).4(\sum \frac{1}{b+c})$
ta chứng minh BDT yếu hơn là
$\frac{3.\sum \frac{b+c}{a}}{a+b+c}\geq 4(\sum \frac{1}{b+c})$
chú ý alaf theo AM-GM thì $\sum \frac{b+c}{a}\geq 6\Rightarrow 2\sum \frac{b+c}{a}-3\geq 9$
vế trái của BDT lúc này sẽ lớn hơn $\frac{(a+b+c)(\frac{1}{a}+\frac{1}{b}+\frac{1}{c})+9}{a+b+c}\geq 4\sum \frac{1}{b+c}$
đây chính là DPCM



#355098 Tìm $x$ để $\sqrt x + \sqrt { - x} $ có nghĩa.

Posted by Waiting for you on 18-09-2012 - 18:01 in Đại số

giúp mình bài này với,suy nghĩ mãi không ra:
tìm tập giá trị của x để tồn tại
mong các bác giúp đỡ :wacko: :wacko: :wacko:

đề ảo thật cỉ có 1 giá trị x=0 thỏa mãn f(x) xác định,bạn giải theo định nghĩa căn xác định khi biểu thức lấy căn không âm thôi



#355061 giá trị tuyệt đối

Posted by Waiting for you on 18-09-2012 - 13:08 in Bất đẳng thức và cực trị

$\frac{2\left | x \right |}{x^{2}+2}\geq 0$
$\left | \frac{6x}{x^{2}+2} +1\right| \geq \left | \frac{6x}{x^{2}+2} \right |+1\geq 1$
$\left | \frac{x}{x^{2}+2} -2\right| \geq \left | \frac{x}{x^{2}+2} \right |+2\geq 2$
$y\geq 3$
Dấu = khi $x=0$ và t/m bdt có dấu Giá trị tuyệt đối

bài bạn sai ở dòng thứ 2 nhé,ta không có
$\left | a+b \right |\geq \left | a \right |+\left | b \right |$ đâu



#354672 hỏi về tâm tỉ cự trong toán vecto

Posted by Waiting for you on 16-09-2012 - 19:38 in Hình học phẳng

Bạn trừ (1) cho (2) thì có (3) thôi. :icon6: .Mà hình như vế phải phải là Vetor 0 chứ nhỉ?

cảm ơn bạn nhiều còn cái dáu kia nữa,ai chi mình với???????????



#354629 hỏi về tâm tỉ cự trong toán vecto

Posted by Waiting for you on 16-09-2012 - 17:19 in Hình học phẳng

hỏi về tâm tỉ cự trong toán vecto
mọi người chỉ cho em cái
VD20 tài liệu chuyên toán hh 10:tam giác ABC có trong tâm G ,M bấtkì,A',B',C' dối xứng với M qua trung điểm BC,CA,AB chứng minh AA',BB',CC' đồng quy tại trung điểm mõi đoạn
lời giải trong TL như sau
GỌI K lầ tâm tỉ cự của hệ(A,B,C,M) với hệ số(1,1,1,-1) từ đẳng thức
$\vec{KA}+\vec{KB}+\vec{KC}-\vec{KM}=0$(1)
CHÚ Ý
$\vec{A'B}+\vec{A'C}+-\vec{A'M}=0$(2)
SUY RA (theo công thức thu gọn)
$\vec{KA}+(1+1-1)\vec{KA'}=0$(3)
CHO EM HỎI TAI SAO TỪ (1)VÀ(2)lại suy ra (3)
(sẵn cho em hỏi luôn $\sum_{i=1}^{n}$ có nghĩa là gì,có phải nhất thiết phải là i=1 không,nếu i= số khác thì sao)
em xin cảm ơn